LSAT and Law School Admissions Forum

Get expert LSAT preparation and law school admissions advice from PowerScore Test Preparation.

 Administrator
PowerScore Staff
  • PowerScore Staff
  • Posts: 8915
  • Joined: Feb 02, 2011
|
#32465
Complete Question Explanation

Assumption. The correct answer choice is (B)

Here, the author disputes the accuracy of certain statements made by several nations in 1997 concerning their oil reserves. Those nations asserted that their oil reserves had not changed in the past year, since the end of 1996. The author, however, concludes that most of the nations were probably mistaken in these assertions.

In reaching this conclusion, the author relies on causal reasoning and probabilistic evidence. We are told that “oil reserves are unlikely to remain unchanged from one year to the next,” because oil is extracted from existing oil fields, reducing reserves, while new oil fields are discovered, increasing oil reserves. Essentially, draining oil and finding new oil fields are both causes of changing oil reserves. These causes are distinct, though they are not mutually exclusive, and could occur at the same time.

Essentially, the author argues that since it is unlikely that oil reserves remain unchanged from one year to the next, then for most of these countries, the reserves probably changed in some way. And, although the author does not explicitly say so, since a change in oil reserves is caused by either draining an existing oil field or discovering a new one, the author must believe that one or both of these causes occurred.

The question stem identifies this as an Assumption question. Our prephrase is that the author assumes that, in most of the nations at issue in 1997, either existing oil fields were drained, new oil fields were discovered, or both causes occurred. If neither of these causes occurred, there would be no basis for the author’s conclusion that these nations are probably incorrect.

Answer choice (A): This answer choice is too expansive, because it refers to any nation. The stimulus dealt only with most of the several nations that stated their oil reserves remain unchanged.

Answer choice (B): This is the correct answer choice, for the reasons described above. If no existing oil fields were drained and no new fields were discovered, the conclusion would be entirely without support.

Answer choice (C): The conclusion does not require that any one nation experience both causes. Instead, it requires that most of the nations experienced as least one of the changes.

Answer choice (D): A counterpart to answer choice (C), this choice is incorrect, because it is not required that both causes occur for a nation’s reserves to change.

Answer choice (E): Refreshingly, this answer choice appears to be an attempt at LSAT humor. The argument has nothing to do with a nation’s duty to accurately report changes in its reserves.
 lsat2016
  • Posts: 59
  • Joined: May 29, 2016
|
#26031
Hello,

I eliminated answer C because it talks about “at least one nation” which is a “some” statement. However, I have learned that you cannot combine a “MOST of the nations are probably incorrect” with a “SOME” statement. Is this a valid method for eliminating the answer choice even though the argument is not based upon conditional logic?

Thank you,
 Clay Cooper
PowerScore Staff
  • PowerScore Staff
  • Posts: 241
  • Joined: Jul 03, 2015
|
#26058
Hi Lsat 2016,

Thanks for your question.

I am not sure I quite understand the question as you have phrased it; let me explain why answer choice C is wrong, and then let me know if that answers your question. Answer choice C is incorrect because it does not have to be true for our conclusion to be true. In other words, it is possible that no nation's oil reserves both gradually declined and rose sharply (as mentioned in C) and yet the conclusion is correct. That means that the negation technique reveals this answer choice to be a bad one; in its negated form, it is not an attack on the conclusion.

I don't think that the 'at least one' phrasing compared to the use of 'most' is as important as the degree of certainty - we see 'probably' in the stimulus, paralleled by 'likely' in answer choice B, and NOT paralleled by the certainty with which C is stated.

I hope that clears it up. If I haven't answered your question, please let me know.
 lsat2016
  • Posts: 59
  • Joined: May 29, 2016
|
#26142
Hello,

Thank you for your response! I certainly agree with you reasoning.
I have learned that you cannot combine a “some” with a “most” statement, but I am not sure when to apply this rule. I just learned this rule by memorization and have yet to apply in a logical reasoning question, and I thought this rule was applicable in this situation because the passage contains a "most" statement and the answer choice contains a "some" statement.

"So MOST of the nations that say so are incorrect"

"In 1997, at least one (SOME) nation changed"

We don't know what happens when we combine a most + some answer choice and therefore this answer is wrong.

I think my thinking for approaching this rule is incorrect. Please let me know where I'm going wrong!

Thank you so much!!
 Nikki Siclunov
PowerScore Staff
  • PowerScore Staff
  • Posts: 1362
  • Joined: Aug 02, 2011
|
#26157
Hi lsat2016,

Thanks for the follow-up. A word of caution is in order: You should never, ever "memorize" any strategies or rules without properly understanding the context in which they would be most helpful. This can cause more harm than good, as you clearly saw yourself here.

The rule against combing "some" and "most" statements pertains to Formal Logic propositions whereby an inference is incorrectly drawn from combining a "some" and a "most" statement:
  • Some snakes are pets. (Snakes :some: Pets)

    Most pets are quite lovely. (Pets :most: Lovely)

    Therefore, some snakes are quite lovely. (Snakes :some: Lovely)
The conclusion is clearly flawed, because most, but not necessarily all, pets are lovely. So, it is entirely possible that the pet snakes are not among the lovely pets :)

The rule against combing "some" and "most" statements has no relevance to the question at hand.

Hope this clears it up!

Thanks,
 bli2016
  • Posts: 67
  • Joined: Nov 29, 2016
|
#33853
Hi, I originally prephrased the answer as something along the lines of "in most of the nations that stated that their oil reserves were unchanged, old oil fields were drained or new oil fields were discover, but not both". The reason I prephrased it that way is because I thought if both occurred, then the oil reserves would be unchanged (i.e., the discovery of the new oil fields might have compensated for the amount drained), so some of the nations could actually be correct in reporting no changes, thereby weakening the author's argument. I eventually chose B through the process of elimination, but I was confused by why the answer was different from my prephrase and still have not figured it out. Would appreciate some help on this end. Thanks!
 Francis O'Rourke
PowerScore Staff
  • PowerScore Staff
  • Posts: 471
  • Joined: Mar 10, 2017
|
#33865
Hi Bli,

You missed one small detail about the math here. If a country drains one oil field and opens another it isn't guaranteed that the level of oil reserves is going to stay the same. In fact, the stimulus tells us that when an oil field is opened, the reserve levels rise suddenly.

Let me know if I'm wrong, but I believe that you are assuming that each new oil field would compensate exactly for each drained oil field.
 NeverMissing
  • Posts: 35
  • Joined: Feb 21, 2017
|
#34339
Hi, I'd like to piggyback off bli2016's response because my prephrase was exactly the same; either the countries drained old reserves or discovered new reserves, but not both.

Francis' response notes that the stimulus tells us that old oil fields drain reserves gradually and new oil fields increase reserves suddenly. I noted this distinction in the stimulus and was sure it would come into play, but I could still foresee a scenario where, even with this premise, overall oil reserves had not changed from the previous year. I'd like help understanding why the premise does not allow for the following hypothetical (which it seems would make answer choice B no longer a strengthener).

Oil reserves for country X at end of 1996: 100 barrels

Jan 1997
Gradually use 5 barrels from old oil field (95 barrels remaining)

Feb 1997
Gradually use 5 barrels from old oil field (90 barrels remaining)

March 1997
Gradually use 5 barrels from old oil field (85 barrels remaining)

April 1997
Gradually use 5 barrels from old oil field (80 barrels remaining)

May 1997
Gradually use 5 barrels from old oil field (75 barrels remaining)

June 1997
Gradually use 5 barrels from old oil field (70 barrels remaining)

July 1997
Gradually use 5 barrels from old oil field (65 barrels remaining)

Aug 1997
Gradually use 5 barrels from old oil field (60 barrels remaining)

Sep 1997
Gradually use 5 barrels from old oil field (55 barrels remaining)

Oct 1997
Gradually use 5 barrels from old oil field (50 barrels remaining)

Nov 1997
Gradually use 5 barrels from old oil field (45 barrels remaining)

Dec 1997
Gradually use 5 barrels from old oil field (40 barrels remaining)
DISCOVER NEW OIL FIELD that contains 60 barrels (The reserves jump suddenly back to 100 barrels)

Oil reserves for country X at end of 1997: 100 barrels (unchanged from 1996 reserves)


The scenario above allows for old oil fields to drain reserves gradually and new oil fields to increase reserves suddenly and still come out unchanged from 1996 to 1997. This is by no means guaranteed, but since it is a possibility allowed for by answer choice B, doesn't this weaken its ability to be a strengthener for the argument?
 Francis O'Rourke
PowerScore Staff
  • PowerScore Staff
  • Posts: 471
  • Joined: Mar 10, 2017
|
#34398
The conclusion states that it is probably incorrect to assume that these nations had unchanged oil reserves. You described one very specific event, which the speaker seems to be accounting for by using uncertain, probabilistic language. When we are discussing discovering millions of barrels of new oil, would you not agree that the situation you described is unlikely?
 Lsat180Please
  • Posts: 44
  • Joined: Sep 12, 2018
|
#58414
I see why C is wrong for a few reasons but I originally eliminated C because it said at least one nation and not "some" or "most". If C for example said "during the course of 1997 it is likely that in at least one nation their oil fields were drained or new oil fields were discovered" would that be correct? I would think no because if only one nation did, her conclusion still does not follow. I guess I want to be sure that I am understanding "most" "some" and "at least one" correctly. Thank you!

Get the most out of your LSAT Prep Plus subscription.

Analyze and track your performance with our Testing and Analytics Package.